Integrating (4x+1)^1/2 | Homework Help

Click For Summary
The integral of (4x+1)^(1/2) was evaluated, with the initial attempt yielding (4x+1)^(3/2)/6. However, the discussion clarified that this result is indeed correct, as differentiating it returns to the original function. Participants emphasized the importance of including the constant of integration, which is often overlooked. The final consensus confirms the accuracy of the solution while reminding to add the constant. The integration process was successfully validated through differentiation.
Priyadarshini
Messages
191
Reaction score
4

Homework Statement


Integrate (4x+1)^1/2

Homework Equations


Integration (ax+b)^n dx= (ax+b)^(n+1)/ a(n+1)

The Attempt at a Solution


(4x+1)^(3/2)/ 4(3/2)
= (4x+1)^(3/2)/6

but the actual answer is
3(4x+1)^(3/2)/8
 
Physics news on Phys.org
Priyadarshini said:

Homework Statement


Integrate (4x+1)^1/2

Homework Equations


Integration (ax+b)^n dx= (ax+b)^(n+1)/ a(n+1)

The Attempt at a Solution


(4x+1)^(3/2)/ 4(3/2)
= (4x+1)^(3/2)/6

but the actual answer is
3(4x+1)^(3/2)/8
Take the derivative of your result and the correct result. Compare to help you see where you went awry .
 
Priyadarshini said:

Homework Statement


Integrate (4x+1)^1/2

Homework Equations


Integration (ax+b)^n dx= (ax+b)^(n+1)/ a(n+1)

The Attempt at a Solution


(4x+1)^(3/2)/ 4(3/2)
= (4x+1)^(3/2)/6

but the actual answer is
3(4x+1)^(3/2)/8

No, the actual answer is what YOU obtained. Try differentiating your answer, to check if you get back your initial ##(4x+1)^{1/2}##.
 
Priyadarshini said:

The Attempt at a Solution


(4x+1)^(3/2)/ 4(3/2)
= (4x+1)^(3/2)/6
Your result is correct. (but do not forget adding a C constant :smile:)
 
Ray Vickson said:
No, the actual answer is what YOU obtained. Try differentiating your answer, to check if you get back your initial ##(4x+1)^{1/2}##.
It does. Thank you!
 
ehild said:
Your result is correct. (but do not forget adding a C constant :smile:)
I keep forgetting to add the constant! Thanks!
 
Question: A clock's minute hand has length 4 and its hour hand has length 3. What is the distance between the tips at the moment when it is increasing most rapidly?(Putnam Exam Question) Answer: Making assumption that both the hands moves at constant angular velocities, the answer is ## \sqrt{7} .## But don't you think this assumption is somewhat doubtful and wrong?

Similar threads

  • · Replies 10 ·
Replies
10
Views
2K
  • · Replies 5 ·
Replies
5
Views
1K
  • · Replies 6 ·
Replies
6
Views
2K
Replies
7
Views
2K
  • · Replies 8 ·
Replies
8
Views
2K
  • · Replies 14 ·
Replies
14
Views
1K
  • · Replies 2 ·
Replies
2
Views
2K
  • · Replies 3 ·
Replies
3
Views
2K
  • · Replies 9 ·
Replies
9
Views
2K
  • · Replies 1 ·
Replies
1
Views
2K